Difference between revisions of "2012 AMC 10B Problems/Problem 6"

m (fixed latex)
m (fixed latex)
Line 3: Line 3:
 
In order to estimate the value of <math>x-y</math> where <math>x</math> and <math>y</math> are real numbers with <math>x > y > 0</math>, Xiaoli rounded <math>x</math> up by a small amount, rounded <math>y</math> down by the same amount, and then subtracted her rounded values. Which of the following statements is necessarily correct?  
 
In order to estimate the value of <math>x-y</math> where <math>x</math> and <math>y</math> are real numbers with <math>x > y > 0</math>, Xiaoli rounded <math>x</math> up by a small amount, rounded <math>y</math> down by the same amount, and then subtracted her rounded values. Which of the following statements is necessarily correct?  
  
<math>\textbf{(A)}\ \text{Her estimate is larger than } x-y \qquad \textbf{(B)}\ \text{Her estimate is smaller than } x-y \qquad \textbf{(C)}\ \text{Her estimate equals } x-y \qquad \textbf{(D)}\ \text{Her estimate equals } x-y \qquad \textbf{(E)}\ \text{Her estimate is } 0</math>
+
<math>\textbf{(A) }</math> Her estimate is larger than <math>x-y \qquad \textbf{(B) }</math> Her estimate is smaller than <math> x-y \qquad \textbf{(C) }</math> Her estimate equals <math>x-y \qquad \textbf{(D) } </math>Her estimate equals <math>x-y \qquad \textbf{(E) }</math> Her estimate is <math>0</math>
  
 
== Solution ==
 
== Solution ==

Revision as of 15:59, 18 January 2020

Problem

In order to estimate the value of $x-y$ where $x$ and $y$ are real numbers with $x > y > 0$, Xiaoli rounded $x$ up by a small amount, rounded $y$ down by the same amount, and then subtracted her rounded values. Which of the following statements is necessarily correct?

$\textbf{(A) }$ Her estimate is larger than $x-y \qquad \textbf{(B) }$ Her estimate is smaller than $x-y \qquad \textbf{(C) }$ Her estimate equals $x-y \qquad \textbf{(D) }$Her estimate equals $x-y \qquad \textbf{(E) }$ Her estimate is $0$

Solution

Let's define $z$ as the amount rounded up by and down by.

The problem statement tells us that Xiaoli performed the following computation:

$\left(x+z\right) - \left(y-z\right) = x+z-y+z = x-y+2z$

We can see that $x-y+2z$ is greater than $x-y$, and so the answer is $\textbf{(A)} \text{Her estimate is larger than } x-y$.


See Also

2012 AMC 10B (ProblemsAnswer KeyResources)
Preceded by
Problem 5
Followed by
Problem 7
1 2 3 4 5 6 7 8 9 10 11 12 13 14 15 16 17 18 19 20 21 22 23 24 25
All AMC 10 Problems and Solutions

The problems on this page are copyrighted by the Mathematical Association of America's American Mathematics Competitions. AMC logo.png